(11 intermediate revisions by 2 users not shown)
Line 15: Line 15:
 
== Question  ==
 
== Question  ==
  
'''Problem 1. ''' (50 pts)
+
Question is posted from this [https://engineering.purdue.edu/ECE/Academics/Graduates/Archived_QE_August_13/CS-5.pdf <u>link</u>].<br>
  
<br>  
+
'''Problem 1. ''' (50 pts) <br>  
  
'''Problem 2. ''' (50 pts)  
+
Consider the 2D discrete space signal&nbsp;<span class="texhtml">''x''(''m'',''n'') with the DSFT of&nbsp;<span class="texhtml">''X''(''e''<sup>''j''μ</sup>,''e''<sup>''j''ν</sup>) given by&nbsp;</span></span>
  
Let&nbsp;'''<span class="texhtml">''r''<sub>0</sub>(λ), &nbsp;<span class="texhtml">''g''<sub>0</sub>(λ)</span></span>'''&nbsp;,&nbsp;<span style="line-height: 19.9200000762939px;">and&nbsp;</span>''b''<sub style="font-family: serif; line-height: 1.5em;">0</sub><span style="font-family: serif; line-height: 1.5em;">(λ)</span><span style="line-height: 19.9200000762939px;">&nbsp;be the CIE color matching functions for red, green, and blue primaries at 700 nm, 546.1 nm, and 435.8 nm, respectively, and let <span class="texhtml">[''r'',''g'',''b'']</span>&nbsp;be the corresponding CIE tristimulus values.&nbsp;</span>
+
<math>X(e^{j\mu},e^{j\nu}) = \sum_{m=-\infty}^{\infty} \sum_{n=-\infty}^{\infty}
 +
x(m,n)e^{-j(m\mu+n\nu)}</math>
 +
 
 +
Then define
 +
 
 +
<math>p_{0}(n) = \sum_{m=-\infty}^{\infty}x(m,n)</math><br>
 +
 
 +
<math>p_{1}(m) = \sum_{n=-\infty}^{\infty}x(m,n)</math>
 +
 
 +
with corresponding DTFT given by&nbsp;
 +
 
 +
<math>P_{0}(e^{j\omega}) = \sum_{n=-\infty}^{\infty} p_{0}(n)e^{-jn\omega}</math><br>
 +
 
 +
<math>P_{1}(e^{j\omega}) = \sum_{m=-\infty}^{\infty} p_{0}(m)e^{-jm\omega}</math><br> a) Derive an expression for&nbsp;<span class="texhtml">''P''<sub>0</sub>(''e''<sup>''j''ω</sup>)</span>&nbsp;in terms of&nbsp;<span class="texhtml">''X''(''e''<sup>''j''μ</sup>,''w''<sup>''j''ν</sup>)</span>.
 +
 
 +
b) Derive an expression&nbsp;<span class="texhtml">''P''<sub>0</sub>(''e''<sup>''j''ω</sup>)</span>&nbsp;in terms of&nbsp;<span class="texhtml">''X''(''e''<sup>''j''μ</sup>,''e''<sup>''j''ν</sup>)</span>.
 +
 
 +
c) Derive an expression &nbsp;for&nbsp;<math>\sum_{n = -\infty}^{\infty}p_0(n)</math>&nbsp;interms of&nbsp;<span class="texhtml">''X''(''e''<sup>''j''μ</sup>,''e''<sup>''j''ν</sup>)</span>.  
 +
 
 +
d) Do the function&nbsp;<span class="texhtml">''p''<sub>0</sub>(''n'')</span>&nbsp;and&nbsp;<span class="texhtml">''p''<sub>1</sub>(''m'')</span>&nbsp;together contains sufficient information to reconstruction the function&nbsp;<span class="texhtml">''x''(''m'',''n'')</span>? If so, provide a reconstruction algorithm; if not, provide a counter example.
 +
 
 +
Click [[QE637 2013 Pro1|here]] to view student [[QE637 2013 Pro1|answers and discussions]] <br>
 +
 
 +
----
 +
 
 +
<br> '''Problem 2. ''' (50 pts)
 +
 
 +
Let <span class="texhtml">''r''<sub>0</sub>(λ)</span>, <span class="texhtml">''g''<sub>0</sub>(λ)</span>, and <span class="texhtml">''b''<sub>0</sub>(λ)</span> be the CIE color matching functions for red, green, and blue primaries at 700 nm, 546.1 nm, and 435.8 nm, respectively, and let <span class="texhtml">[''r'',''g'',''b'']</span>&nbsp;be the corresponding CIE tristimulus values.&nbsp;&lt;/span&gt;
  
 
Furthermore, let&nbsp;<span class="texhtml">''f''<sub>1</sub>(λ)</span>,&nbsp;<span class="texhtml">''f''<sub>2</sub>(λ)</span>, and&nbsp;<span class="texhtml">''f''<sub>3</sub>(λ)</span>&nbsp;be the spectral response functions for the three color outputs of a color camera. So for each pixel of the camera sensor, there is a 3-dimensional output vector given by&nbsp;<span class="texhtml">''F'' = [''F''<sub>1</sub>,''F''<sub>2</sub>,''F''<sub>3</sub>]<sup>''t''</sup></span>, where  
 
Furthermore, let&nbsp;<span class="texhtml">''f''<sub>1</sub>(λ)</span>,&nbsp;<span class="texhtml">''f''<sub>2</sub>(λ)</span>, and&nbsp;<span class="texhtml">''f''<sub>3</sub>(λ)</span>&nbsp;be the spectral response functions for the three color outputs of a color camera. So for each pixel of the camera sensor, there is a 3-dimensional output vector given by&nbsp;<span class="texhtml">''F'' = [''F''<sub>1</sub>,''F''<sub>2</sub>,''F''<sub>3</sub>]<sup>''t''</sup></span>, where  
Line 47: Line 74:
 
b_0(\lambda)
 
b_0(\lambda)
 
\end{array}} \right]
 
\end{array}} \right]
</math>
+
</math>  
  
 +
<br> a) Why is it necessary that&nbsp;<math>f_k(\lambda) \geq 0</math>&nbsp;for&nbsp;<span class="texhtml">''k'' = 0,1,2</span>?
  
a) Why is it necessary that&nbsp;<math>f_k(\lambda) \geq 0</math>&nbsp;for&nbsp;<span class="texhtml">''k'' = 0,1,2</span>?&lt;span style="line-height: 1.5em;" /&gt;
+
b) Are the functions, <math> r_0(\lambda) \geq 0</math>, <math>g_0(\lambda) \geq 0</math>, and <math>b_0(\lambda) \geq 0</math>? If so, why? If not, why not?
  
b) Are the functions, <math> r_0(\lambda) \geq 0</math>, <math>g_0(\lambda) \geq 0</math>, and <math>b_0(\lambda) \geq 0</math>? If so, why? If not, why not?
+
c) Derive an formula for the tristimulus vector <span class="texhtml">[''r'',''g'',''b'']<sup>''t''</sup></span> in terms of the tristimulus vector <span class="texhtml">''F'' = [''F''<sub>1</sub>,''F''<sub>2</sub>,''F''<sub>3</sub>]<sup>''t''</sup></span>.
  
c) Derive an formula for the tristimulus vector <math>[r, g, b]^t </math> in terms of the tristimulus vector <math> F=[F_1, F_2, F_3]^t </math>.
+
d) Do functions <span class="texhtml">''f''<sub>''k''</sub>(λ)</span> exist, which meet these requirements? If so, give a specific example of such functions.
 +
 
 +
Click [[QE637 2013 Pro2|here]] to view student [[QE637 2013 Pro2|answers and discussions]]
 +
 
 +
<br>  
  
d) Do functions <math> f_k(\lambda) </math> exist, which meet these requirements? If so, give a specific example of such functions.
 
 
[[Category:ECE]] [[Category:QE]] [[Category:CNSIP]] [[Category:Problem_solving]] [[Category:Image_processing]]
 
[[Category:ECE]] [[Category:QE]] [[Category:CNSIP]] [[Category:Problem_solving]] [[Category:Image_processing]]

Latest revision as of 18:06, 12 November 2014


ECE Ph.D. Qualifying Exam

Communication, Networking, Signal and Image Processing (CS)

Question 5: Image Processing

August 2013



Question

Question is posted from this link.

Problem 1. (50 pts)

Consider the 2D discrete space signal x(m,n) with the DSFT of X(ejμ,ejν) given by 

$ X(e^{j\mu},e^{j\nu}) = \sum_{m=-\infty}^{\infty} \sum_{n=-\infty}^{\infty} x(m,n)e^{-j(m\mu+n\nu)} $

Then define

$ p_{0}(n) = \sum_{m=-\infty}^{\infty}x(m,n) $

$ p_{1}(m) = \sum_{n=-\infty}^{\infty}x(m,n) $

with corresponding DTFT given by 

$ P_{0}(e^{j\omega}) = \sum_{n=-\infty}^{\infty} p_{0}(n)e^{-jn\omega} $

$ P_{1}(e^{j\omega}) = \sum_{m=-\infty}^{\infty} p_{0}(m)e^{-jm\omega} $
a) Derive an expression for P0(ejω) in terms of X(ejμ,wjν).

b) Derive an expression P0(ejω) in terms of X(ejμ,ejν).

c) Derive an expression  for $ \sum_{n = -\infty}^{\infty}p_0(n) $ interms of X(ejμ,ejν).

d) Do the function p0(n) and p1(m) together contains sufficient information to reconstruction the function x(m,n)? If so, provide a reconstruction algorithm; if not, provide a counter example.

Click here to view student answers and discussions



Problem 2. (50 pts)

Let r0(λ), g0(λ), and b0(λ) be the CIE color matching functions for red, green, and blue primaries at 700 nm, 546.1 nm, and 435.8 nm, respectively, and let [r,g,b] be the corresponding CIE tristimulus values. </span>

Furthermore, let f1(λ)f2(λ), and f3(λ) be the spectral response functions for the three color outputs of a color camera. So for each pixel of the camera sensor, there is a 3-dimensional output vector given by F = [F1,F2,F3]t, where

$ F_1 = \int_{-\infty}^{\infty}f_1(\lambda)I(\lambda)d\lambda $,

$ F_2 = \int_{-\infty}^{\infty}f_2(\lambda)I(\lambda)d\lambda $,

$ F_3 = \int_{-\infty}^{\infty}f_3(\lambda)I(\lambda)d\lambda $

where I(λ) is the energy spectrum of the incoming light and $ f_k(\lambda)\geq 0 $ for k = 0,1,2..

Furthermore, assume there exists a matrix, M, so that

$ \left[ {\begin{array}{*{20}{c}} f_1(\lambda)\\ f_1(\lambda)\\ f_1(\lambda) \end{array}} \right] = {\begin{array}{*{20}{c}} M \end{array}} \left[ {\begin{array}{*{20}{c}} r_0(\lambda)\\ g_0(\lambda)\\ b_0(\lambda) \end{array}} \right] $


a) Why is it necessary that $ f_k(\lambda) \geq 0 $ for k = 0,1,2?

b) Are the functions, $ r_0(\lambda) \geq 0 $, $ g_0(\lambda) \geq 0 $, and $ b_0(\lambda) \geq 0 $? If so, why? If not, why not?

c) Derive an formula for the tristimulus vector [r,g,b]t in terms of the tristimulus vector F = [F1,F2,F3]t.

d) Do functions fk(λ) exist, which meet these requirements? If so, give a specific example of such functions.

Click here to view student answers and discussions


Alumni Liaison

Recent Math PhD now doing a post-doctorate at UC Riverside.

Kuei-Nuan Lin